Which courses Alicia takes is fully determined if she takes Russian and which one of the following?

Milica Gligic on October 23, 2019

I am not sure what this question is asking. Could someone please explain.

Q: Which courses Alicia takes is fully determined if she takes Russian and which one of the following?

Replies
Create a free account to read and take part in forum discussions.

Already have an account? log in

Milica Gligic on October 23, 2019

Never mind got it...The courses Alicia takes are fully determinede if she takes Russian and which ine of the following..

Irina on October 24, 2019

@Milica Gligic,

Let us know if you have any other questions.

Isabel-Zuniga on June 7, 2020

Hi, I have a question about how we ruled our option A so quickly. I thought if we have W then, from the contrapositive of rule 3, S 9am & P would be out?

Vennela-Vellanki on July 19, 2020

^^^I have the same question, how is A not correct?

shunhe on July 27, 2020

Hi @Isabel-Zuniga and @Vennela-Vellanki,

Thanks for the question! So let’s say that we pick Russian and World History together. Will that completely determine the rest of the game, the other two courses?

Well, we have Russian, and we have World History. As you’ve pointed out, this means that we don’t take statistics at 9am. And this means that we don’t take psychology. We also know that she can’t take geography, since she can’t take both geography and world history.

So why are our remaining options? We can still choose from Japanese, Macroeconomics and statistics at 3PM. Are we able to narrow it down further?

Well, let’s say we pick Macroeconomics. Then we can’t pick Japanese. That means she has to take statistics at 3PM.

But what if we don’t pick macroeconomics? Well, she could pick statistics at 3PM and Japanese. So there are actually two possibilities here!

R WH M S3pm

R WH S3pm J

And so Alicia’s courses are completely determined, since there’s two possibilities, and (A) is wrong.

Hope this helps! Feel free to ask any other questions that you might have.

Emil-Kunkin on April 28 at 01:48AM

A is incorrect since we only know that RW are in and G and S9 are out. The contrapositive of rule three tells us nothing about this scenario.